Gravity magic - How does it work?How do I grant superheroes gravitational manipulation, justified scientifically?If magic is real, can it be true that rational scientific thought should exclude it?What would be the impact on a medieval society if “cure disease” magic was available to the powerful?What could hard light be?What kind of spells would magicians cast in combat if the fuel to magic is very expensiveHow would constrained magic affect otherwise high technology warfare?How do I grant superheroes gravitational manipulation, justified scientifically?How would a world full of magic adapt to fighting Mechas?Magic: Reality check of effects of mass-altering magicHow to realistically implement magic-users in medieval warfare?How could my hard telekinesis work?

Is it normal that my co-workers at a fitness company criticize my food choices?

New passport but visa is in old (lost) passport

Instead of a Universal Basic Income program, why not implement a "Universal Basic Needs" program?

If I can solve Sudoku, can I solve the Travelling Salesman Problem (TSP)? If so, how?

Python if-else code style for reduced code for rounding floats

Meme-controlled people

Have researchers managed to "reverse time"? If so, what does that mean for physics?

Violin - Can double stops be played when the strings are not next to each other?

Fastest way to pop N items from a large dict

What options are left, if Britain cannot decide?

Life insurance that covers only simultaneous/dual deaths

Time travel from stationary position?

Shortcut for setting origin to vertex

My adviser wants to be the first author

Why Choose Less Effective Armour Types?

gcc: how to detect bad `bool` usage

Did Ender ever learn that he killed Stilson and/or Bonzo?

How to make healing in an exploration game interesting

Is honey really a supersaturated solution? Does heating to un-crystalize redissolve it or melt it?

Do I need life insurance if I can cover my own funeral costs?

Recruiter wants very extensive technical details about all of my previous work

ERC721: How to get the owned tokens of an address

Are all passive ability checks floors for active ability checks?

Do the common programs (for example: "ls", "cat") in Linux and BSD come from the same source code?



Gravity magic - How does it work?


How do I grant superheroes gravitational manipulation, justified scientifically?If magic is real, can it be true that rational scientific thought should exclude it?What would be the impact on a medieval society if “cure disease” magic was available to the powerful?What could hard light be?What kind of spells would magicians cast in combat if the fuel to magic is very expensiveHow would constrained magic affect otherwise high technology warfare?How do I grant superheroes gravitational manipulation, justified scientifically?How would a world full of magic adapt to fighting Mechas?Magic: Reality check of effects of mass-altering magicHow to realistically implement magic-users in medieval warfare?How could my hard telekinesis work?













15












$begingroup$


This question graduated from the Sandbox for Proposed Questions




Gravity magic - How does it work?



(What physical law can magic manipulate in order to affect gravity?)



In my world everyone is capable of casting magic. It is possible for a single person to cast as many different types of magic as he/she wishes, assuming they know the spells. Every person also has an affinity to a specific type/form of magic, e.g. projectile magic, water magic or defensive magic. One of the main characters in my story has an affinity to gravity-based spells, so it is much easier and more intuitive to use them for her.



What I'm interested in is not how magic works in this context (assume there is some amount of recharging MP within every person that is used to cast spells), but rather how the gravitational effects caused by the magic could be explained.



I'm looking for a "generic" solution that explains explicitly this type of magic, not every single spell/usage, but rather "gravity-type magic" itself. I don't intend to explain my readers every single law of physics, space-time etc. in every detail, but the main aspect of "how" should come across.



When answering, take the following scenarios as use-cases for the magic to check:



  • Slowing down yourself down when falling, so you don't hit the ground at full speed (as near to 0m/s as possible, at the very least you shouldn't be injured by landing) (e.g. jumping from higher up)

  • Levitating yourself / other objects, so they don't move in any direction

    • Bonus points: still won't move if influenced by another force (e.g. a thrown stone)


  • Pushing objects out of your way (e.g. a locked door)

  • Ability to combine multiple "gravity fields", so you could simultaneously keep a cup in place and rotate it upwards-down, while still keeping the liquid inside

I've also found the question How do I grant superheroes gravitational manipulation, justified scientifically?, but it is somewhat different than my question. My question focuses on how to actually perform the gravity-based magic and not on how you could justify the existence of such powers.










share|improve this question









$endgroup$







  • 6




    $begingroup$
    Good job using the sandbox.
    $endgroup$
    – L.Dutch
    yesterday






  • 1




    $begingroup$
    Do note that changing your mass doesn't make you fall any slower unless you also change your buoyancy in air, so magic is in the air! Allez, allez, allez...
    $endgroup$
    – user6760
    yesterday







  • 3




    $begingroup$
    Brandon Sanderson's "The Stormlight Archive" series present a nice example of gravity magic. Aside from being a very good book, it may inspire you.
    $endgroup$
    – Echox
    yesterday






  • 2




    $begingroup$
    @Echox Come for the gravity magic, stay for the bizarrely alien worldbuilding, the setting that manages to make huge fantasy swords and magic power armor actually work, the spren, Shadesmar, the secret societies, the Unmade, the dozens of POV characters, the political intrigue, Wit, the personal drama... take your pick. Stormlight has something for everyone!
    $endgroup$
    – Mason Wheeler
    yesterday
















15












$begingroup$


This question graduated from the Sandbox for Proposed Questions




Gravity magic - How does it work?



(What physical law can magic manipulate in order to affect gravity?)



In my world everyone is capable of casting magic. It is possible for a single person to cast as many different types of magic as he/she wishes, assuming they know the spells. Every person also has an affinity to a specific type/form of magic, e.g. projectile magic, water magic or defensive magic. One of the main characters in my story has an affinity to gravity-based spells, so it is much easier and more intuitive to use them for her.



What I'm interested in is not how magic works in this context (assume there is some amount of recharging MP within every person that is used to cast spells), but rather how the gravitational effects caused by the magic could be explained.



I'm looking for a "generic" solution that explains explicitly this type of magic, not every single spell/usage, but rather "gravity-type magic" itself. I don't intend to explain my readers every single law of physics, space-time etc. in every detail, but the main aspect of "how" should come across.



When answering, take the following scenarios as use-cases for the magic to check:



  • Slowing down yourself down when falling, so you don't hit the ground at full speed (as near to 0m/s as possible, at the very least you shouldn't be injured by landing) (e.g. jumping from higher up)

  • Levitating yourself / other objects, so they don't move in any direction

    • Bonus points: still won't move if influenced by another force (e.g. a thrown stone)


  • Pushing objects out of your way (e.g. a locked door)

  • Ability to combine multiple "gravity fields", so you could simultaneously keep a cup in place and rotate it upwards-down, while still keeping the liquid inside

I've also found the question How do I grant superheroes gravitational manipulation, justified scientifically?, but it is somewhat different than my question. My question focuses on how to actually perform the gravity-based magic and not on how you could justify the existence of such powers.










share|improve this question









$endgroup$







  • 6




    $begingroup$
    Good job using the sandbox.
    $endgroup$
    – L.Dutch
    yesterday






  • 1




    $begingroup$
    Do note that changing your mass doesn't make you fall any slower unless you also change your buoyancy in air, so magic is in the air! Allez, allez, allez...
    $endgroup$
    – user6760
    yesterday







  • 3




    $begingroup$
    Brandon Sanderson's "The Stormlight Archive" series present a nice example of gravity magic. Aside from being a very good book, it may inspire you.
    $endgroup$
    – Echox
    yesterday






  • 2




    $begingroup$
    @Echox Come for the gravity magic, stay for the bizarrely alien worldbuilding, the setting that manages to make huge fantasy swords and magic power armor actually work, the spren, Shadesmar, the secret societies, the Unmade, the dozens of POV characters, the political intrigue, Wit, the personal drama... take your pick. Stormlight has something for everyone!
    $endgroup$
    – Mason Wheeler
    yesterday














15












15








15


1



$begingroup$


This question graduated from the Sandbox for Proposed Questions




Gravity magic - How does it work?



(What physical law can magic manipulate in order to affect gravity?)



In my world everyone is capable of casting magic. It is possible for a single person to cast as many different types of magic as he/she wishes, assuming they know the spells. Every person also has an affinity to a specific type/form of magic, e.g. projectile magic, water magic or defensive magic. One of the main characters in my story has an affinity to gravity-based spells, so it is much easier and more intuitive to use them for her.



What I'm interested in is not how magic works in this context (assume there is some amount of recharging MP within every person that is used to cast spells), but rather how the gravitational effects caused by the magic could be explained.



I'm looking for a "generic" solution that explains explicitly this type of magic, not every single spell/usage, but rather "gravity-type magic" itself. I don't intend to explain my readers every single law of physics, space-time etc. in every detail, but the main aspect of "how" should come across.



When answering, take the following scenarios as use-cases for the magic to check:



  • Slowing down yourself down when falling, so you don't hit the ground at full speed (as near to 0m/s as possible, at the very least you shouldn't be injured by landing) (e.g. jumping from higher up)

  • Levitating yourself / other objects, so they don't move in any direction

    • Bonus points: still won't move if influenced by another force (e.g. a thrown stone)


  • Pushing objects out of your way (e.g. a locked door)

  • Ability to combine multiple "gravity fields", so you could simultaneously keep a cup in place and rotate it upwards-down, while still keeping the liquid inside

I've also found the question How do I grant superheroes gravitational manipulation, justified scientifically?, but it is somewhat different than my question. My question focuses on how to actually perform the gravity-based magic and not on how you could justify the existence of such powers.










share|improve this question









$endgroup$




This question graduated from the Sandbox for Proposed Questions




Gravity magic - How does it work?



(What physical law can magic manipulate in order to affect gravity?)



In my world everyone is capable of casting magic. It is possible for a single person to cast as many different types of magic as he/she wishes, assuming they know the spells. Every person also has an affinity to a specific type/form of magic, e.g. projectile magic, water magic or defensive magic. One of the main characters in my story has an affinity to gravity-based spells, so it is much easier and more intuitive to use them for her.



What I'm interested in is not how magic works in this context (assume there is some amount of recharging MP within every person that is used to cast spells), but rather how the gravitational effects caused by the magic could be explained.



I'm looking for a "generic" solution that explains explicitly this type of magic, not every single spell/usage, but rather "gravity-type magic" itself. I don't intend to explain my readers every single law of physics, space-time etc. in every detail, but the main aspect of "how" should come across.



When answering, take the following scenarios as use-cases for the magic to check:



  • Slowing down yourself down when falling, so you don't hit the ground at full speed (as near to 0m/s as possible, at the very least you shouldn't be injured by landing) (e.g. jumping from higher up)

  • Levitating yourself / other objects, so they don't move in any direction

    • Bonus points: still won't move if influenced by another force (e.g. a thrown stone)


  • Pushing objects out of your way (e.g. a locked door)

  • Ability to combine multiple "gravity fields", so you could simultaneously keep a cup in place and rotate it upwards-down, while still keeping the liquid inside

I've also found the question How do I grant superheroes gravitational manipulation, justified scientifically?, but it is somewhat different than my question. My question focuses on how to actually perform the gravity-based magic and not on how you could justify the existence of such powers.







science-based magic gravity






share|improve this question













share|improve this question











share|improve this question




share|improve this question










asked yesterday









Tobias F.Tobias F.

1767




1767







  • 6




    $begingroup$
    Good job using the sandbox.
    $endgroup$
    – L.Dutch
    yesterday






  • 1




    $begingroup$
    Do note that changing your mass doesn't make you fall any slower unless you also change your buoyancy in air, so magic is in the air! Allez, allez, allez...
    $endgroup$
    – user6760
    yesterday







  • 3




    $begingroup$
    Brandon Sanderson's "The Stormlight Archive" series present a nice example of gravity magic. Aside from being a very good book, it may inspire you.
    $endgroup$
    – Echox
    yesterday






  • 2




    $begingroup$
    @Echox Come for the gravity magic, stay for the bizarrely alien worldbuilding, the setting that manages to make huge fantasy swords and magic power armor actually work, the spren, Shadesmar, the secret societies, the Unmade, the dozens of POV characters, the political intrigue, Wit, the personal drama... take your pick. Stormlight has something for everyone!
    $endgroup$
    – Mason Wheeler
    yesterday













  • 6




    $begingroup$
    Good job using the sandbox.
    $endgroup$
    – L.Dutch
    yesterday






  • 1




    $begingroup$
    Do note that changing your mass doesn't make you fall any slower unless you also change your buoyancy in air, so magic is in the air! Allez, allez, allez...
    $endgroup$
    – user6760
    yesterday







  • 3




    $begingroup$
    Brandon Sanderson's "The Stormlight Archive" series present a nice example of gravity magic. Aside from being a very good book, it may inspire you.
    $endgroup$
    – Echox
    yesterday






  • 2




    $begingroup$
    @Echox Come for the gravity magic, stay for the bizarrely alien worldbuilding, the setting that manages to make huge fantasy swords and magic power armor actually work, the spren, Shadesmar, the secret societies, the Unmade, the dozens of POV characters, the political intrigue, Wit, the personal drama... take your pick. Stormlight has something for everyone!
    $endgroup$
    – Mason Wheeler
    yesterday








6




6




$begingroup$
Good job using the sandbox.
$endgroup$
– L.Dutch
yesterday




$begingroup$
Good job using the sandbox.
$endgroup$
– L.Dutch
yesterday




1




1




$begingroup$
Do note that changing your mass doesn't make you fall any slower unless you also change your buoyancy in air, so magic is in the air! Allez, allez, allez...
$endgroup$
– user6760
yesterday





$begingroup$
Do note that changing your mass doesn't make you fall any slower unless you also change your buoyancy in air, so magic is in the air! Allez, allez, allez...
$endgroup$
– user6760
yesterday





3




3




$begingroup$
Brandon Sanderson's "The Stormlight Archive" series present a nice example of gravity magic. Aside from being a very good book, it may inspire you.
$endgroup$
– Echox
yesterday




$begingroup$
Brandon Sanderson's "The Stormlight Archive" series present a nice example of gravity magic. Aside from being a very good book, it may inspire you.
$endgroup$
– Echox
yesterday




2




2




$begingroup$
@Echox Come for the gravity magic, stay for the bizarrely alien worldbuilding, the setting that manages to make huge fantasy swords and magic power armor actually work, the spren, Shadesmar, the secret societies, the Unmade, the dozens of POV characters, the political intrigue, Wit, the personal drama... take your pick. Stormlight has something for everyone!
$endgroup$
– Mason Wheeler
yesterday





$begingroup$
@Echox Come for the gravity magic, stay for the bizarrely alien worldbuilding, the setting that manages to make huge fantasy swords and magic power armor actually work, the spren, Shadesmar, the secret societies, the Unmade, the dozens of POV characters, the political intrigue, Wit, the personal drama... take your pick. Stormlight has something for everyone!
$endgroup$
– Mason Wheeler
yesterday











8 Answers
8






active

oldest

votes


















10












$begingroup$


What I'm interested in is not how magic works in this context (assume there is some amount of recharging MP within every person that is used to cast spells), but rather how the gravitational effects caused by the magic could be explained.




If you stick to Einstein's model for gravity, you can explain your magic via the ability of manipulating the curvature of space-time.



The Gravitational force experienced by an object (the Einstein tensor) subject to curvature $R$ (the Ricci tensor) can be calculated by the formula $G_µv = $$8πG over C^4$$ T_µv$



To create a "repulsion" effect, the curvature can be made convex instead of concave. To limit how large an area is affected, use a sharper curve (this will cause the force experience to increase / decrease more rapidly as you approach the peak, instead of the normal $r^-2$ relationship experienced at distance $r$ under standard conditions)



By curving space-time directly, you can also apply the force in any direction instead of just "towards" and "away"






share|improve this answer









$endgroup$








  • 1




    $begingroup$
    As for how that magic works, you change the curvature by changing the population of virtual particles in the intervening space: Changing their density changes c, and changing their dielectric changes G.
    $endgroup$
    – amI
    14 hours ago






  • 3




    $begingroup$
    @amI Do you mean $C$? Changing $c$ would be concerning.
    $endgroup$
    – wizzwizz4
    8 hours ago


















7












$begingroup$


What I'm interested in is not how magic works in this context (assume there is some amount of recharging MP within every person that is used to cast spells), but rather how the gravitational effects caused by the magic could be explained.




If you stitch to the Newtonian model for gravity, you can explain your magic via the ability of controlling magnitude and sign of the masses involved in the gravitational interaction.



The gravitational force between two bodies of mass m at distance r can be calculated according to the famous formula $F=G cdot$$m_1 cdot m_2 over r^2$.



If you can change the sign of m, you can have gravity be repulsive. So, in case you jump from a skyscraper, turning your mass to negative once in fall would have the effect of Earth pushing you up, slowing down your fall. You can also levitate by alternating positive and negative mass values.



Turning upside down a cup while not dropping its content could be achieved in the same way.






share|improve this answer









$endgroup$








  • 1




    $begingroup$
    One thing to consider is that gravity is ridiculously weak compared to any other force. So if you limit the amount you can change the magnitude and sign, you will likely only be able to either crush things or levitate them, if the effect is uniform, since you are always pushing against the earth. Nonuniform gravity can induce some spin in objects, but it will be really hard to generate any sideways motion
    $endgroup$
    – Whitecold
    yesterday











  • $begingroup$
    Turning your mass to negative would have the effect of you pushing Earth down. Earth would also push you up, but if your inertial mass was also negative that would result in your moving down. (If you're just changing your gravitational mass without changing your inertial mass, you're very strange.)
    $endgroup$
    – wizzwizz4
    8 hours ago


















5












$begingroup$

Personally I'd steer clear of trying to get some physics-based in-universe explanation unless your magical society also has a very high technology.



That said...



Since gravity is the result of curvature of space, the simple answer would be to have the magic operate to alter that curvature. We don't know exactly what mechanism causes space to curve in the presence of mass, so you can get away with some artistic licence. It's magic after all.



Let's say that the gravitational field strength, which is an expression of the warping of space, is something that you can move around. You can't make much gravity on your own, but you can potentially take energy from other parts of the field and concentrate it. Reducing the gravity in an area requires you to spread that field energy around, so gravity gets stronger elsewhere. The better you are at controlling it the more complex you can make the resulting changes, and the stronger your magical talent the larger the scope of the changes.



But gravity only really does one thing: pull on stuff. You can put a gravity well in the air 2 feet above the table and watch it fall in, but controlling the orientation of the table is going to be difficult. You'd probably need to create a shaped gravity field the shape of the table to get it in the right orientation, and it's not just going to sit there when you push on it... unless you have enough field strength to counter the push.



Since this is magic though, let's assume that you can deform space the way you want if you have the right spell. You could do all sorts of nasty things with it, like concentrating all of the gravity in a battlefield into a tiny ball of hyper-gravity strong enough to tear humans apart. You could make things fall in any direction, essentially making you the ultimate siege weapon. Water flowing uphill is way simple. How about castles falling into the sky? If it were my castle I'd be doing my best to outlaw gravity magic, as aggressively as possible.



So yes, you can stop people falling. Yes, you can walk on walls and ceilings. But it's going to take a hell of a lot of field shaping to get cups to dance and flip without spilling their contents.



Perhaps what you need is less gravity and more telekinesis? Directly applying forces to objects has a lot of potential too. Maybe a combination of both?






share|improve this answer









$endgroup$












  • $begingroup$
    We don't know, +1
    $endgroup$
    – Mazura
    21 hours ago


















3












$begingroup$

Gravity sources are everywhere. Everything in the galaxy is attracting everything else.

The only reason the satellites in space don't come crashing down is because they are traveling fast enough to keep missing the Earth.

The only reason the Earth doesn't crash into the sun is that it's moving fast enough to keep missing it.

The only reason the sun doesn't fall into the black hole at the galactic core is that it's moving fast enough to keep missing it.



If any of them were to lose forward momentum they'd plunge to a fiery death.



The only reason two apples don't orbit each other is because they are overwhelmed by earths gravity, and the suns gravity, and all of the other gravity sources that influence us.



So the way that magic controls gravity is by selectively ignoring, reversing, or reinforcing it around whatever thing you want to manipulate. If you need to levitate, reverse the pull of Earths gravity. If you need to push something, reinforce the pull from things in line with the direction you want it to go, and then negate or reverse the pull from the other directions.



At the highest levels it would be crazy powerful, being able to increase an items gravity field to the point where it becomes neutronium or a singularity.






share|improve this answer









$endgroup$




















    2












    $begingroup$

    The kind of magic you are talking about is anthropomorphic.




    • Slowing down yourself down when falling, so you don't hit the ground at full speed (as near to 0m/s as possible, at the very least you shouldn't be injured by landing) (e.g. jumping from higher up)

    • Levitating yourself / other objects, so they don't move in any direction

    • Bonus points: still won't move if influenced by another force (e.g. a thrown stone)

    • Pushing objects out of your way (e.g. a locked door)

    • Ability to combine multiple "gravity fields", so you could simultaneously keep a cup in place and rotate it upwards-down, while still keeping the liquid inside



    Gravity is the measurement of the degree to which mass-energy curves space time around it. What you describe above is a bunch of stuff that seems convincing to a human if you described gravity magic to.



    A physics based process won't neatly fit in a "stuff that X magic would do" that a human would narrate, because physics doesn't really care what a human finds narratively convincing.



    As an example, Electro-Magnitism covers everything from being able to touch things, see things, shoot lightning, and pretty much all of chemistry. Meanwhile, the human "story" based Magnatism is about being able to move metals around (because they are "magnetic"), and Electricty magic is about lightning bolts and static electricty.



    If you want anthropomorphic magic, magic that makes sense mythologically or in categories humans care about, it has to be powered by an intelligence with a similar mind to ours. That intelligence somehow set up the rules in ways that we find are pleasing.



    Spells become "instructions" to this intelligence, who then does what asked. The being being interacted with could be a god-like entity, a demon, or a post-singularity AI, or an ancient forgotten society of super-scientists who left "macros" active that their children used to play with.



    In the last case, the affinity of certain people for certain kinds of magic is akin to being able to unlock a certain smart phone with your voice, because it coincidentally is similar to the real owner (it doesn't have to be voice-locked, but you can imagine it being so). The fact that these ancient intelligences let their children play with fireballs would be explained by the fact that everyone in their society was backed up, and repair/resurrection was trivial; that technology doesn't recognize humans as being in their pattern-banks. Humans who can heal are actually using the "veternarian" macros.



    Now, once we divorce the physical mechanism from the "kind of magic", the fact that these are "gravity" macros is because they are the kind of thing that the parent programmed for their kid (or maybe bought the "play with gravity" kit for their kid's system).



    Your entire environment could be a simulation running in an ancient computer, with the original hosts long exinct. The matter you are walking on and breathing could be programmable computronium (mass-energy optimized to do computation) that emulates being matter so as to leave a "natural" experience for growing up, but being computronium this is just an option; "magic" literally ignores the apparent laws of physics, because the laws of physics as you experience them are just an emulation layer.



    Being able to float in either of these cases is little more than a matter of changing the position value of a variable in a specific coordinate space. Now, no human has access to this underlying system; so they cannot insert the wrong arguments and cause the water on the earth to start to orbit the other way around the sun.






    share|improve this answer









    $endgroup$












    • $begingroup$
      "There is no spoon"
      $endgroup$
      – EveryBitHelps
      12 hours ago


















    2












    $begingroup$

    I'm borrowing heavily from the "Irregular at the Magic Highschool" light novel series where magic is done by influencing the information bodies of particles. Basically assume everything has an "information body" similar to how in physics simulations every particle has some sort of matrix/vector which stores its properties. These properties can be position, time, velocity, mass, energy, etc... This is somewhat similar to string theory where particles are strings and properties of a particle are determined by the vibration of the string.



    Whichever way you want to go about it, gravity magic influences the vibration/information of the particle related to gravity. There are no "gravity fields" in this solution so I can't check off the ability to create multiple gravity fields but you can still rotate a cup and not have any liquid fall out by influencing the particles of the cup along with the liquid inside.






    share|improve this answer









    $endgroup$




















      1












      $begingroup$

      This is more of an addition to L.Dutch♦'s answer, controlling magnitude and sign of the masses involved in the gravitational interaction, but to achieve some of the points you mentioned (pushing a door out of the way, stopping movement even when affected by an outside force) you need to have a solution for sideways motion and rotation.



      For sideways motion you can take into account the fact that Earth isn't a point source for gravity, instead we are pulled towards each particle that makes up the Earth and our gravity vector is the sum of all those forces. If your character can control the effect of gravity from different sources separately, then sideways motion can be achieved by getting a push from one half of the Earth and getting pulled by the other. You could even use gravity from other celestial bodies (Sun, Jupiter, supermassive black hole at the center of the the Milky Way).



      Rotation can be achieved as described in Whitecold's comment, with nonuniform gravity: have half of the thing you're rotating be pulled down and the other half pulled up (gently so you don't tear it apart). Your rotating cup of water that doesn't spill could be done by having gravity push "into" the cup just near the mouth, pushing from the bottom of the cup to counteract the movement the first push would impart, all the while keeping the cup and water in the air (this might require more skill from your character than simple levitation).






      share|improve this answer








      New contributor




      JCP is a new contributor to this site. Take care in asking for clarification, commenting, and answering.
      Check out our Code of Conduct.






      $endgroup$




















        1












        $begingroup$

        The Higgs field gives elementary particles their mass, and having some sort of a mechanism to attenuate or disable its interactions could work to your benefit at a level that doesn't really have a reasonably obvious counterargument. So in effect, the magic could somehow stop this interaction, making object momentarily more or less massive. (The remote pushing effect you mentioned I found the most difficult to explain...) The intricacies work very much differently for charged leptons and force carrier bosons, though.



        Most of the mass of matter is from the mass of the atomic nuclei, and ≈99% of that mass doesn't come from the mass of the individual elementary particles but the strong force; potential energies of the quarks (and hence protons and neutrons) bound together can be translated to mass via E=mc^2. Another explanation for your magic could thus be not exactly one that works by meddling with gravity but meddling with another fundamental force which affects mass which in turn gets picked up unmodified gravity. The strong force is mediated by gluons (analogous to photons for the electromagnetic interactions), so off we go with gluemancy.



        The problem with this all is of course, that if one is capable enough to modify these tine, fundamental properties of particles, it doesn't easily make much sense to be able to only demonstrate gravity-based powers.






        share|improve this answer








        New contributor




        JoonasD6 is a new contributor to this site. Take care in asking for clarification, commenting, and answering.
        Check out our Code of Conduct.






        $endgroup$












          Your Answer





          StackExchange.ifUsing("editor", function ()
          return StackExchange.using("mathjaxEditing", function ()
          StackExchange.MarkdownEditor.creationCallbacks.add(function (editor, postfix)
          StackExchange.mathjaxEditing.prepareWmdForMathJax(editor, postfix, [["$", "$"], ["\\(","\\)"]]);
          );
          );
          , "mathjax-editing");

          StackExchange.ready(function()
          var channelOptions =
          tags: "".split(" "),
          id: "579"
          ;
          initTagRenderer("".split(" "), "".split(" "), channelOptions);

          StackExchange.using("externalEditor", function()
          // Have to fire editor after snippets, if snippets enabled
          if (StackExchange.settings.snippets.snippetsEnabled)
          StackExchange.using("snippets", function()
          createEditor();
          );

          else
          createEditor();

          );

          function createEditor()
          StackExchange.prepareEditor(
          heartbeatType: 'answer',
          autoActivateHeartbeat: false,
          convertImagesToLinks: false,
          noModals: true,
          showLowRepImageUploadWarning: true,
          reputationToPostImages: null,
          bindNavPrevention: true,
          postfix: "",
          imageUploader:
          brandingHtml: "Powered by u003ca class="icon-imgur-white" href="https://imgur.com/"u003eu003c/au003e",
          contentPolicyHtml: "User contributions licensed under u003ca href="https://creativecommons.org/licenses/by-sa/3.0/"u003ecc by-sa 3.0 with attribution requiredu003c/au003e u003ca href="https://stackoverflow.com/legal/content-policy"u003e(content policy)u003c/au003e",
          allowUrls: true
          ,
          noCode: true, onDemand: true,
          discardSelector: ".discard-answer"
          ,immediatelyShowMarkdownHelp:true
          );



          );













          draft saved

          draft discarded


















          StackExchange.ready(
          function ()
          StackExchange.openid.initPostLogin('.new-post-login', 'https%3a%2f%2fworldbuilding.stackexchange.com%2fquestions%2f141533%2fgravity-magic-how-does-it-work%23new-answer', 'question_page');

          );

          Post as a guest















          Required, but never shown

























          8 Answers
          8






          active

          oldest

          votes








          8 Answers
          8






          active

          oldest

          votes









          active

          oldest

          votes






          active

          oldest

          votes









          10












          $begingroup$


          What I'm interested in is not how magic works in this context (assume there is some amount of recharging MP within every person that is used to cast spells), but rather how the gravitational effects caused by the magic could be explained.




          If you stick to Einstein's model for gravity, you can explain your magic via the ability of manipulating the curvature of space-time.



          The Gravitational force experienced by an object (the Einstein tensor) subject to curvature $R$ (the Ricci tensor) can be calculated by the formula $G_µv = $$8πG over C^4$$ T_µv$



          To create a "repulsion" effect, the curvature can be made convex instead of concave. To limit how large an area is affected, use a sharper curve (this will cause the force experience to increase / decrease more rapidly as you approach the peak, instead of the normal $r^-2$ relationship experienced at distance $r$ under standard conditions)



          By curving space-time directly, you can also apply the force in any direction instead of just "towards" and "away"






          share|improve this answer









          $endgroup$








          • 1




            $begingroup$
            As for how that magic works, you change the curvature by changing the population of virtual particles in the intervening space: Changing their density changes c, and changing their dielectric changes G.
            $endgroup$
            – amI
            14 hours ago






          • 3




            $begingroup$
            @amI Do you mean $C$? Changing $c$ would be concerning.
            $endgroup$
            – wizzwizz4
            8 hours ago















          10












          $begingroup$


          What I'm interested in is not how magic works in this context (assume there is some amount of recharging MP within every person that is used to cast spells), but rather how the gravitational effects caused by the magic could be explained.




          If you stick to Einstein's model for gravity, you can explain your magic via the ability of manipulating the curvature of space-time.



          The Gravitational force experienced by an object (the Einstein tensor) subject to curvature $R$ (the Ricci tensor) can be calculated by the formula $G_µv = $$8πG over C^4$$ T_µv$



          To create a "repulsion" effect, the curvature can be made convex instead of concave. To limit how large an area is affected, use a sharper curve (this will cause the force experience to increase / decrease more rapidly as you approach the peak, instead of the normal $r^-2$ relationship experienced at distance $r$ under standard conditions)



          By curving space-time directly, you can also apply the force in any direction instead of just "towards" and "away"






          share|improve this answer









          $endgroup$








          • 1




            $begingroup$
            As for how that magic works, you change the curvature by changing the population of virtual particles in the intervening space: Changing their density changes c, and changing their dielectric changes G.
            $endgroup$
            – amI
            14 hours ago






          • 3




            $begingroup$
            @amI Do you mean $C$? Changing $c$ would be concerning.
            $endgroup$
            – wizzwizz4
            8 hours ago













          10












          10








          10





          $begingroup$


          What I'm interested in is not how magic works in this context (assume there is some amount of recharging MP within every person that is used to cast spells), but rather how the gravitational effects caused by the magic could be explained.




          If you stick to Einstein's model for gravity, you can explain your magic via the ability of manipulating the curvature of space-time.



          The Gravitational force experienced by an object (the Einstein tensor) subject to curvature $R$ (the Ricci tensor) can be calculated by the formula $G_µv = $$8πG over C^4$$ T_µv$



          To create a "repulsion" effect, the curvature can be made convex instead of concave. To limit how large an area is affected, use a sharper curve (this will cause the force experience to increase / decrease more rapidly as you approach the peak, instead of the normal $r^-2$ relationship experienced at distance $r$ under standard conditions)



          By curving space-time directly, you can also apply the force in any direction instead of just "towards" and "away"






          share|improve this answer









          $endgroup$




          What I'm interested in is not how magic works in this context (assume there is some amount of recharging MP within every person that is used to cast spells), but rather how the gravitational effects caused by the magic could be explained.




          If you stick to Einstein's model for gravity, you can explain your magic via the ability of manipulating the curvature of space-time.



          The Gravitational force experienced by an object (the Einstein tensor) subject to curvature $R$ (the Ricci tensor) can be calculated by the formula $G_µv = $$8πG over C^4$$ T_µv$



          To create a "repulsion" effect, the curvature can be made convex instead of concave. To limit how large an area is affected, use a sharper curve (this will cause the force experience to increase / decrease more rapidly as you approach the peak, instead of the normal $r^-2$ relationship experienced at distance $r$ under standard conditions)



          By curving space-time directly, you can also apply the force in any direction instead of just "towards" and "away"







          share|improve this answer












          share|improve this answer



          share|improve this answer










          answered yesterday









          ChronocidalChronocidal

          6,4431832




          6,4431832







          • 1




            $begingroup$
            As for how that magic works, you change the curvature by changing the population of virtual particles in the intervening space: Changing their density changes c, and changing their dielectric changes G.
            $endgroup$
            – amI
            14 hours ago






          • 3




            $begingroup$
            @amI Do you mean $C$? Changing $c$ would be concerning.
            $endgroup$
            – wizzwizz4
            8 hours ago












          • 1




            $begingroup$
            As for how that magic works, you change the curvature by changing the population of virtual particles in the intervening space: Changing their density changes c, and changing their dielectric changes G.
            $endgroup$
            – amI
            14 hours ago






          • 3




            $begingroup$
            @amI Do you mean $C$? Changing $c$ would be concerning.
            $endgroup$
            – wizzwizz4
            8 hours ago







          1




          1




          $begingroup$
          As for how that magic works, you change the curvature by changing the population of virtual particles in the intervening space: Changing their density changes c, and changing their dielectric changes G.
          $endgroup$
          – amI
          14 hours ago




          $begingroup$
          As for how that magic works, you change the curvature by changing the population of virtual particles in the intervening space: Changing their density changes c, and changing their dielectric changes G.
          $endgroup$
          – amI
          14 hours ago




          3




          3




          $begingroup$
          @amI Do you mean $C$? Changing $c$ would be concerning.
          $endgroup$
          – wizzwizz4
          8 hours ago




          $begingroup$
          @amI Do you mean $C$? Changing $c$ would be concerning.
          $endgroup$
          – wizzwizz4
          8 hours ago











          7












          $begingroup$


          What I'm interested in is not how magic works in this context (assume there is some amount of recharging MP within every person that is used to cast spells), but rather how the gravitational effects caused by the magic could be explained.




          If you stitch to the Newtonian model for gravity, you can explain your magic via the ability of controlling magnitude and sign of the masses involved in the gravitational interaction.



          The gravitational force between two bodies of mass m at distance r can be calculated according to the famous formula $F=G cdot$$m_1 cdot m_2 over r^2$.



          If you can change the sign of m, you can have gravity be repulsive. So, in case you jump from a skyscraper, turning your mass to negative once in fall would have the effect of Earth pushing you up, slowing down your fall. You can also levitate by alternating positive and negative mass values.



          Turning upside down a cup while not dropping its content could be achieved in the same way.






          share|improve this answer









          $endgroup$








          • 1




            $begingroup$
            One thing to consider is that gravity is ridiculously weak compared to any other force. So if you limit the amount you can change the magnitude and sign, you will likely only be able to either crush things or levitate them, if the effect is uniform, since you are always pushing against the earth. Nonuniform gravity can induce some spin in objects, but it will be really hard to generate any sideways motion
            $endgroup$
            – Whitecold
            yesterday











          • $begingroup$
            Turning your mass to negative would have the effect of you pushing Earth down. Earth would also push you up, but if your inertial mass was also negative that would result in your moving down. (If you're just changing your gravitational mass without changing your inertial mass, you're very strange.)
            $endgroup$
            – wizzwizz4
            8 hours ago















          7












          $begingroup$


          What I'm interested in is not how magic works in this context (assume there is some amount of recharging MP within every person that is used to cast spells), but rather how the gravitational effects caused by the magic could be explained.




          If you stitch to the Newtonian model for gravity, you can explain your magic via the ability of controlling magnitude and sign of the masses involved in the gravitational interaction.



          The gravitational force between two bodies of mass m at distance r can be calculated according to the famous formula $F=G cdot$$m_1 cdot m_2 over r^2$.



          If you can change the sign of m, you can have gravity be repulsive. So, in case you jump from a skyscraper, turning your mass to negative once in fall would have the effect of Earth pushing you up, slowing down your fall. You can also levitate by alternating positive and negative mass values.



          Turning upside down a cup while not dropping its content could be achieved in the same way.






          share|improve this answer









          $endgroup$








          • 1




            $begingroup$
            One thing to consider is that gravity is ridiculously weak compared to any other force. So if you limit the amount you can change the magnitude and sign, you will likely only be able to either crush things or levitate them, if the effect is uniform, since you are always pushing against the earth. Nonuniform gravity can induce some spin in objects, but it will be really hard to generate any sideways motion
            $endgroup$
            – Whitecold
            yesterday











          • $begingroup$
            Turning your mass to negative would have the effect of you pushing Earth down. Earth would also push you up, but if your inertial mass was also negative that would result in your moving down. (If you're just changing your gravitational mass without changing your inertial mass, you're very strange.)
            $endgroup$
            – wizzwizz4
            8 hours ago













          7












          7








          7





          $begingroup$


          What I'm interested in is not how magic works in this context (assume there is some amount of recharging MP within every person that is used to cast spells), but rather how the gravitational effects caused by the magic could be explained.




          If you stitch to the Newtonian model for gravity, you can explain your magic via the ability of controlling magnitude and sign of the masses involved in the gravitational interaction.



          The gravitational force between two bodies of mass m at distance r can be calculated according to the famous formula $F=G cdot$$m_1 cdot m_2 over r^2$.



          If you can change the sign of m, you can have gravity be repulsive. So, in case you jump from a skyscraper, turning your mass to negative once in fall would have the effect of Earth pushing you up, slowing down your fall. You can also levitate by alternating positive and negative mass values.



          Turning upside down a cup while not dropping its content could be achieved in the same way.






          share|improve this answer









          $endgroup$




          What I'm interested in is not how magic works in this context (assume there is some amount of recharging MP within every person that is used to cast spells), but rather how the gravitational effects caused by the magic could be explained.




          If you stitch to the Newtonian model for gravity, you can explain your magic via the ability of controlling magnitude and sign of the masses involved in the gravitational interaction.



          The gravitational force between two bodies of mass m at distance r can be calculated according to the famous formula $F=G cdot$$m_1 cdot m_2 over r^2$.



          If you can change the sign of m, you can have gravity be repulsive. So, in case you jump from a skyscraper, turning your mass to negative once in fall would have the effect of Earth pushing you up, slowing down your fall. You can also levitate by alternating positive and negative mass values.



          Turning upside down a cup while not dropping its content could be achieved in the same way.







          share|improve this answer












          share|improve this answer



          share|improve this answer










          answered yesterday









          L.DutchL.Dutch

          87.7k29205427




          87.7k29205427







          • 1




            $begingroup$
            One thing to consider is that gravity is ridiculously weak compared to any other force. So if you limit the amount you can change the magnitude and sign, you will likely only be able to either crush things or levitate them, if the effect is uniform, since you are always pushing against the earth. Nonuniform gravity can induce some spin in objects, but it will be really hard to generate any sideways motion
            $endgroup$
            – Whitecold
            yesterday











          • $begingroup$
            Turning your mass to negative would have the effect of you pushing Earth down. Earth would also push you up, but if your inertial mass was also negative that would result in your moving down. (If you're just changing your gravitational mass without changing your inertial mass, you're very strange.)
            $endgroup$
            – wizzwizz4
            8 hours ago












          • 1




            $begingroup$
            One thing to consider is that gravity is ridiculously weak compared to any other force. So if you limit the amount you can change the magnitude and sign, you will likely only be able to either crush things or levitate them, if the effect is uniform, since you are always pushing against the earth. Nonuniform gravity can induce some spin in objects, but it will be really hard to generate any sideways motion
            $endgroup$
            – Whitecold
            yesterday











          • $begingroup$
            Turning your mass to negative would have the effect of you pushing Earth down. Earth would also push you up, but if your inertial mass was also negative that would result in your moving down. (If you're just changing your gravitational mass without changing your inertial mass, you're very strange.)
            $endgroup$
            – wizzwizz4
            8 hours ago







          1




          1




          $begingroup$
          One thing to consider is that gravity is ridiculously weak compared to any other force. So if you limit the amount you can change the magnitude and sign, you will likely only be able to either crush things or levitate them, if the effect is uniform, since you are always pushing against the earth. Nonuniform gravity can induce some spin in objects, but it will be really hard to generate any sideways motion
          $endgroup$
          – Whitecold
          yesterday





          $begingroup$
          One thing to consider is that gravity is ridiculously weak compared to any other force. So if you limit the amount you can change the magnitude and sign, you will likely only be able to either crush things or levitate them, if the effect is uniform, since you are always pushing against the earth. Nonuniform gravity can induce some spin in objects, but it will be really hard to generate any sideways motion
          $endgroup$
          – Whitecold
          yesterday













          $begingroup$
          Turning your mass to negative would have the effect of you pushing Earth down. Earth would also push you up, but if your inertial mass was also negative that would result in your moving down. (If you're just changing your gravitational mass without changing your inertial mass, you're very strange.)
          $endgroup$
          – wizzwizz4
          8 hours ago




          $begingroup$
          Turning your mass to negative would have the effect of you pushing Earth down. Earth would also push you up, but if your inertial mass was also negative that would result in your moving down. (If you're just changing your gravitational mass without changing your inertial mass, you're very strange.)
          $endgroup$
          – wizzwizz4
          8 hours ago











          5












          $begingroup$

          Personally I'd steer clear of trying to get some physics-based in-universe explanation unless your magical society also has a very high technology.



          That said...



          Since gravity is the result of curvature of space, the simple answer would be to have the magic operate to alter that curvature. We don't know exactly what mechanism causes space to curve in the presence of mass, so you can get away with some artistic licence. It's magic after all.



          Let's say that the gravitational field strength, which is an expression of the warping of space, is something that you can move around. You can't make much gravity on your own, but you can potentially take energy from other parts of the field and concentrate it. Reducing the gravity in an area requires you to spread that field energy around, so gravity gets stronger elsewhere. The better you are at controlling it the more complex you can make the resulting changes, and the stronger your magical talent the larger the scope of the changes.



          But gravity only really does one thing: pull on stuff. You can put a gravity well in the air 2 feet above the table and watch it fall in, but controlling the orientation of the table is going to be difficult. You'd probably need to create a shaped gravity field the shape of the table to get it in the right orientation, and it's not just going to sit there when you push on it... unless you have enough field strength to counter the push.



          Since this is magic though, let's assume that you can deform space the way you want if you have the right spell. You could do all sorts of nasty things with it, like concentrating all of the gravity in a battlefield into a tiny ball of hyper-gravity strong enough to tear humans apart. You could make things fall in any direction, essentially making you the ultimate siege weapon. Water flowing uphill is way simple. How about castles falling into the sky? If it were my castle I'd be doing my best to outlaw gravity magic, as aggressively as possible.



          So yes, you can stop people falling. Yes, you can walk on walls and ceilings. But it's going to take a hell of a lot of field shaping to get cups to dance and flip without spilling their contents.



          Perhaps what you need is less gravity and more telekinesis? Directly applying forces to objects has a lot of potential too. Maybe a combination of both?






          share|improve this answer









          $endgroup$












          • $begingroup$
            We don't know, +1
            $endgroup$
            – Mazura
            21 hours ago















          5












          $begingroup$

          Personally I'd steer clear of trying to get some physics-based in-universe explanation unless your magical society also has a very high technology.



          That said...



          Since gravity is the result of curvature of space, the simple answer would be to have the magic operate to alter that curvature. We don't know exactly what mechanism causes space to curve in the presence of mass, so you can get away with some artistic licence. It's magic after all.



          Let's say that the gravitational field strength, which is an expression of the warping of space, is something that you can move around. You can't make much gravity on your own, but you can potentially take energy from other parts of the field and concentrate it. Reducing the gravity in an area requires you to spread that field energy around, so gravity gets stronger elsewhere. The better you are at controlling it the more complex you can make the resulting changes, and the stronger your magical talent the larger the scope of the changes.



          But gravity only really does one thing: pull on stuff. You can put a gravity well in the air 2 feet above the table and watch it fall in, but controlling the orientation of the table is going to be difficult. You'd probably need to create a shaped gravity field the shape of the table to get it in the right orientation, and it's not just going to sit there when you push on it... unless you have enough field strength to counter the push.



          Since this is magic though, let's assume that you can deform space the way you want if you have the right spell. You could do all sorts of nasty things with it, like concentrating all of the gravity in a battlefield into a tiny ball of hyper-gravity strong enough to tear humans apart. You could make things fall in any direction, essentially making you the ultimate siege weapon. Water flowing uphill is way simple. How about castles falling into the sky? If it were my castle I'd be doing my best to outlaw gravity magic, as aggressively as possible.



          So yes, you can stop people falling. Yes, you can walk on walls and ceilings. But it's going to take a hell of a lot of field shaping to get cups to dance and flip without spilling their contents.



          Perhaps what you need is less gravity and more telekinesis? Directly applying forces to objects has a lot of potential too. Maybe a combination of both?






          share|improve this answer









          $endgroup$












          • $begingroup$
            We don't know, +1
            $endgroup$
            – Mazura
            21 hours ago













          5












          5








          5





          $begingroup$

          Personally I'd steer clear of trying to get some physics-based in-universe explanation unless your magical society also has a very high technology.



          That said...



          Since gravity is the result of curvature of space, the simple answer would be to have the magic operate to alter that curvature. We don't know exactly what mechanism causes space to curve in the presence of mass, so you can get away with some artistic licence. It's magic after all.



          Let's say that the gravitational field strength, which is an expression of the warping of space, is something that you can move around. You can't make much gravity on your own, but you can potentially take energy from other parts of the field and concentrate it. Reducing the gravity in an area requires you to spread that field energy around, so gravity gets stronger elsewhere. The better you are at controlling it the more complex you can make the resulting changes, and the stronger your magical talent the larger the scope of the changes.



          But gravity only really does one thing: pull on stuff. You can put a gravity well in the air 2 feet above the table and watch it fall in, but controlling the orientation of the table is going to be difficult. You'd probably need to create a shaped gravity field the shape of the table to get it in the right orientation, and it's not just going to sit there when you push on it... unless you have enough field strength to counter the push.



          Since this is magic though, let's assume that you can deform space the way you want if you have the right spell. You could do all sorts of nasty things with it, like concentrating all of the gravity in a battlefield into a tiny ball of hyper-gravity strong enough to tear humans apart. You could make things fall in any direction, essentially making you the ultimate siege weapon. Water flowing uphill is way simple. How about castles falling into the sky? If it were my castle I'd be doing my best to outlaw gravity magic, as aggressively as possible.



          So yes, you can stop people falling. Yes, you can walk on walls and ceilings. But it's going to take a hell of a lot of field shaping to get cups to dance and flip without spilling their contents.



          Perhaps what you need is less gravity and more telekinesis? Directly applying forces to objects has a lot of potential too. Maybe a combination of both?






          share|improve this answer









          $endgroup$



          Personally I'd steer clear of trying to get some physics-based in-universe explanation unless your magical society also has a very high technology.



          That said...



          Since gravity is the result of curvature of space, the simple answer would be to have the magic operate to alter that curvature. We don't know exactly what mechanism causes space to curve in the presence of mass, so you can get away with some artistic licence. It's magic after all.



          Let's say that the gravitational field strength, which is an expression of the warping of space, is something that you can move around. You can't make much gravity on your own, but you can potentially take energy from other parts of the field and concentrate it. Reducing the gravity in an area requires you to spread that field energy around, so gravity gets stronger elsewhere. The better you are at controlling it the more complex you can make the resulting changes, and the stronger your magical talent the larger the scope of the changes.



          But gravity only really does one thing: pull on stuff. You can put a gravity well in the air 2 feet above the table and watch it fall in, but controlling the orientation of the table is going to be difficult. You'd probably need to create a shaped gravity field the shape of the table to get it in the right orientation, and it's not just going to sit there when you push on it... unless you have enough field strength to counter the push.



          Since this is magic though, let's assume that you can deform space the way you want if you have the right spell. You could do all sorts of nasty things with it, like concentrating all of the gravity in a battlefield into a tiny ball of hyper-gravity strong enough to tear humans apart. You could make things fall in any direction, essentially making you the ultimate siege weapon. Water flowing uphill is way simple. How about castles falling into the sky? If it were my castle I'd be doing my best to outlaw gravity magic, as aggressively as possible.



          So yes, you can stop people falling. Yes, you can walk on walls and ceilings. But it's going to take a hell of a lot of field shaping to get cups to dance and flip without spilling their contents.



          Perhaps what you need is less gravity and more telekinesis? Directly applying forces to objects has a lot of potential too. Maybe a combination of both?







          share|improve this answer












          share|improve this answer



          share|improve this answer










          answered yesterday









          CoreyCorey

          944411




          944411











          • $begingroup$
            We don't know, +1
            $endgroup$
            – Mazura
            21 hours ago
















          • $begingroup$
            We don't know, +1
            $endgroup$
            – Mazura
            21 hours ago















          $begingroup$
          We don't know, +1
          $endgroup$
          – Mazura
          21 hours ago




          $begingroup$
          We don't know, +1
          $endgroup$
          – Mazura
          21 hours ago











          3












          $begingroup$

          Gravity sources are everywhere. Everything in the galaxy is attracting everything else.

          The only reason the satellites in space don't come crashing down is because they are traveling fast enough to keep missing the Earth.

          The only reason the Earth doesn't crash into the sun is that it's moving fast enough to keep missing it.

          The only reason the sun doesn't fall into the black hole at the galactic core is that it's moving fast enough to keep missing it.



          If any of them were to lose forward momentum they'd plunge to a fiery death.



          The only reason two apples don't orbit each other is because they are overwhelmed by earths gravity, and the suns gravity, and all of the other gravity sources that influence us.



          So the way that magic controls gravity is by selectively ignoring, reversing, or reinforcing it around whatever thing you want to manipulate. If you need to levitate, reverse the pull of Earths gravity. If you need to push something, reinforce the pull from things in line with the direction you want it to go, and then negate or reverse the pull from the other directions.



          At the highest levels it would be crazy powerful, being able to increase an items gravity field to the point where it becomes neutronium or a singularity.






          share|improve this answer









          $endgroup$

















            3












            $begingroup$

            Gravity sources are everywhere. Everything in the galaxy is attracting everything else.

            The only reason the satellites in space don't come crashing down is because they are traveling fast enough to keep missing the Earth.

            The only reason the Earth doesn't crash into the sun is that it's moving fast enough to keep missing it.

            The only reason the sun doesn't fall into the black hole at the galactic core is that it's moving fast enough to keep missing it.



            If any of them were to lose forward momentum they'd plunge to a fiery death.



            The only reason two apples don't orbit each other is because they are overwhelmed by earths gravity, and the suns gravity, and all of the other gravity sources that influence us.



            So the way that magic controls gravity is by selectively ignoring, reversing, or reinforcing it around whatever thing you want to manipulate. If you need to levitate, reverse the pull of Earths gravity. If you need to push something, reinforce the pull from things in line with the direction you want it to go, and then negate or reverse the pull from the other directions.



            At the highest levels it would be crazy powerful, being able to increase an items gravity field to the point where it becomes neutronium or a singularity.






            share|improve this answer









            $endgroup$















              3












              3








              3





              $begingroup$

              Gravity sources are everywhere. Everything in the galaxy is attracting everything else.

              The only reason the satellites in space don't come crashing down is because they are traveling fast enough to keep missing the Earth.

              The only reason the Earth doesn't crash into the sun is that it's moving fast enough to keep missing it.

              The only reason the sun doesn't fall into the black hole at the galactic core is that it's moving fast enough to keep missing it.



              If any of them were to lose forward momentum they'd plunge to a fiery death.



              The only reason two apples don't orbit each other is because they are overwhelmed by earths gravity, and the suns gravity, and all of the other gravity sources that influence us.



              So the way that magic controls gravity is by selectively ignoring, reversing, or reinforcing it around whatever thing you want to manipulate. If you need to levitate, reverse the pull of Earths gravity. If you need to push something, reinforce the pull from things in line with the direction you want it to go, and then negate or reverse the pull from the other directions.



              At the highest levels it would be crazy powerful, being able to increase an items gravity field to the point where it becomes neutronium or a singularity.






              share|improve this answer









              $endgroup$



              Gravity sources are everywhere. Everything in the galaxy is attracting everything else.

              The only reason the satellites in space don't come crashing down is because they are traveling fast enough to keep missing the Earth.

              The only reason the Earth doesn't crash into the sun is that it's moving fast enough to keep missing it.

              The only reason the sun doesn't fall into the black hole at the galactic core is that it's moving fast enough to keep missing it.



              If any of them were to lose forward momentum they'd plunge to a fiery death.



              The only reason two apples don't orbit each other is because they are overwhelmed by earths gravity, and the suns gravity, and all of the other gravity sources that influence us.



              So the way that magic controls gravity is by selectively ignoring, reversing, or reinforcing it around whatever thing you want to manipulate. If you need to levitate, reverse the pull of Earths gravity. If you need to push something, reinforce the pull from things in line with the direction you want it to go, and then negate or reverse the pull from the other directions.



              At the highest levels it would be crazy powerful, being able to increase an items gravity field to the point where it becomes neutronium or a singularity.







              share|improve this answer












              share|improve this answer



              share|improve this answer










              answered yesterday









              AndyD273AndyD273

              31.1k258137




              31.1k258137





















                  2












                  $begingroup$

                  The kind of magic you are talking about is anthropomorphic.




                  • Slowing down yourself down when falling, so you don't hit the ground at full speed (as near to 0m/s as possible, at the very least you shouldn't be injured by landing) (e.g. jumping from higher up)

                  • Levitating yourself / other objects, so they don't move in any direction

                  • Bonus points: still won't move if influenced by another force (e.g. a thrown stone)

                  • Pushing objects out of your way (e.g. a locked door)

                  • Ability to combine multiple "gravity fields", so you could simultaneously keep a cup in place and rotate it upwards-down, while still keeping the liquid inside



                  Gravity is the measurement of the degree to which mass-energy curves space time around it. What you describe above is a bunch of stuff that seems convincing to a human if you described gravity magic to.



                  A physics based process won't neatly fit in a "stuff that X magic would do" that a human would narrate, because physics doesn't really care what a human finds narratively convincing.



                  As an example, Electro-Magnitism covers everything from being able to touch things, see things, shoot lightning, and pretty much all of chemistry. Meanwhile, the human "story" based Magnatism is about being able to move metals around (because they are "magnetic"), and Electricty magic is about lightning bolts and static electricty.



                  If you want anthropomorphic magic, magic that makes sense mythologically or in categories humans care about, it has to be powered by an intelligence with a similar mind to ours. That intelligence somehow set up the rules in ways that we find are pleasing.



                  Spells become "instructions" to this intelligence, who then does what asked. The being being interacted with could be a god-like entity, a demon, or a post-singularity AI, or an ancient forgotten society of super-scientists who left "macros" active that their children used to play with.



                  In the last case, the affinity of certain people for certain kinds of magic is akin to being able to unlock a certain smart phone with your voice, because it coincidentally is similar to the real owner (it doesn't have to be voice-locked, but you can imagine it being so). The fact that these ancient intelligences let their children play with fireballs would be explained by the fact that everyone in their society was backed up, and repair/resurrection was trivial; that technology doesn't recognize humans as being in their pattern-banks. Humans who can heal are actually using the "veternarian" macros.



                  Now, once we divorce the physical mechanism from the "kind of magic", the fact that these are "gravity" macros is because they are the kind of thing that the parent programmed for their kid (or maybe bought the "play with gravity" kit for their kid's system).



                  Your entire environment could be a simulation running in an ancient computer, with the original hosts long exinct. The matter you are walking on and breathing could be programmable computronium (mass-energy optimized to do computation) that emulates being matter so as to leave a "natural" experience for growing up, but being computronium this is just an option; "magic" literally ignores the apparent laws of physics, because the laws of physics as you experience them are just an emulation layer.



                  Being able to float in either of these cases is little more than a matter of changing the position value of a variable in a specific coordinate space. Now, no human has access to this underlying system; so they cannot insert the wrong arguments and cause the water on the earth to start to orbit the other way around the sun.






                  share|improve this answer









                  $endgroup$












                  • $begingroup$
                    "There is no spoon"
                    $endgroup$
                    – EveryBitHelps
                    12 hours ago















                  2












                  $begingroup$

                  The kind of magic you are talking about is anthropomorphic.




                  • Slowing down yourself down when falling, so you don't hit the ground at full speed (as near to 0m/s as possible, at the very least you shouldn't be injured by landing) (e.g. jumping from higher up)

                  • Levitating yourself / other objects, so they don't move in any direction

                  • Bonus points: still won't move if influenced by another force (e.g. a thrown stone)

                  • Pushing objects out of your way (e.g. a locked door)

                  • Ability to combine multiple "gravity fields", so you could simultaneously keep a cup in place and rotate it upwards-down, while still keeping the liquid inside



                  Gravity is the measurement of the degree to which mass-energy curves space time around it. What you describe above is a bunch of stuff that seems convincing to a human if you described gravity magic to.



                  A physics based process won't neatly fit in a "stuff that X magic would do" that a human would narrate, because physics doesn't really care what a human finds narratively convincing.



                  As an example, Electro-Magnitism covers everything from being able to touch things, see things, shoot lightning, and pretty much all of chemistry. Meanwhile, the human "story" based Magnatism is about being able to move metals around (because they are "magnetic"), and Electricty magic is about lightning bolts and static electricty.



                  If you want anthropomorphic magic, magic that makes sense mythologically or in categories humans care about, it has to be powered by an intelligence with a similar mind to ours. That intelligence somehow set up the rules in ways that we find are pleasing.



                  Spells become "instructions" to this intelligence, who then does what asked. The being being interacted with could be a god-like entity, a demon, or a post-singularity AI, or an ancient forgotten society of super-scientists who left "macros" active that their children used to play with.



                  In the last case, the affinity of certain people for certain kinds of magic is akin to being able to unlock a certain smart phone with your voice, because it coincidentally is similar to the real owner (it doesn't have to be voice-locked, but you can imagine it being so). The fact that these ancient intelligences let their children play with fireballs would be explained by the fact that everyone in their society was backed up, and repair/resurrection was trivial; that technology doesn't recognize humans as being in their pattern-banks. Humans who can heal are actually using the "veternarian" macros.



                  Now, once we divorce the physical mechanism from the "kind of magic", the fact that these are "gravity" macros is because they are the kind of thing that the parent programmed for their kid (or maybe bought the "play with gravity" kit for their kid's system).



                  Your entire environment could be a simulation running in an ancient computer, with the original hosts long exinct. The matter you are walking on and breathing could be programmable computronium (mass-energy optimized to do computation) that emulates being matter so as to leave a "natural" experience for growing up, but being computronium this is just an option; "magic" literally ignores the apparent laws of physics, because the laws of physics as you experience them are just an emulation layer.



                  Being able to float in either of these cases is little more than a matter of changing the position value of a variable in a specific coordinate space. Now, no human has access to this underlying system; so they cannot insert the wrong arguments and cause the water on the earth to start to orbit the other way around the sun.






                  share|improve this answer









                  $endgroup$












                  • $begingroup$
                    "There is no spoon"
                    $endgroup$
                    – EveryBitHelps
                    12 hours ago













                  2












                  2








                  2





                  $begingroup$

                  The kind of magic you are talking about is anthropomorphic.




                  • Slowing down yourself down when falling, so you don't hit the ground at full speed (as near to 0m/s as possible, at the very least you shouldn't be injured by landing) (e.g. jumping from higher up)

                  • Levitating yourself / other objects, so they don't move in any direction

                  • Bonus points: still won't move if influenced by another force (e.g. a thrown stone)

                  • Pushing objects out of your way (e.g. a locked door)

                  • Ability to combine multiple "gravity fields", so you could simultaneously keep a cup in place and rotate it upwards-down, while still keeping the liquid inside



                  Gravity is the measurement of the degree to which mass-energy curves space time around it. What you describe above is a bunch of stuff that seems convincing to a human if you described gravity magic to.



                  A physics based process won't neatly fit in a "stuff that X magic would do" that a human would narrate, because physics doesn't really care what a human finds narratively convincing.



                  As an example, Electro-Magnitism covers everything from being able to touch things, see things, shoot lightning, and pretty much all of chemistry. Meanwhile, the human "story" based Magnatism is about being able to move metals around (because they are "magnetic"), and Electricty magic is about lightning bolts and static electricty.



                  If you want anthropomorphic magic, magic that makes sense mythologically or in categories humans care about, it has to be powered by an intelligence with a similar mind to ours. That intelligence somehow set up the rules in ways that we find are pleasing.



                  Spells become "instructions" to this intelligence, who then does what asked. The being being interacted with could be a god-like entity, a demon, or a post-singularity AI, or an ancient forgotten society of super-scientists who left "macros" active that their children used to play with.



                  In the last case, the affinity of certain people for certain kinds of magic is akin to being able to unlock a certain smart phone with your voice, because it coincidentally is similar to the real owner (it doesn't have to be voice-locked, but you can imagine it being so). The fact that these ancient intelligences let their children play with fireballs would be explained by the fact that everyone in their society was backed up, and repair/resurrection was trivial; that technology doesn't recognize humans as being in their pattern-banks. Humans who can heal are actually using the "veternarian" macros.



                  Now, once we divorce the physical mechanism from the "kind of magic", the fact that these are "gravity" macros is because they are the kind of thing that the parent programmed for their kid (or maybe bought the "play with gravity" kit for their kid's system).



                  Your entire environment could be a simulation running in an ancient computer, with the original hosts long exinct. The matter you are walking on and breathing could be programmable computronium (mass-energy optimized to do computation) that emulates being matter so as to leave a "natural" experience for growing up, but being computronium this is just an option; "magic" literally ignores the apparent laws of physics, because the laws of physics as you experience them are just an emulation layer.



                  Being able to float in either of these cases is little more than a matter of changing the position value of a variable in a specific coordinate space. Now, no human has access to this underlying system; so they cannot insert the wrong arguments and cause the water on the earth to start to orbit the other way around the sun.






                  share|improve this answer









                  $endgroup$



                  The kind of magic you are talking about is anthropomorphic.




                  • Slowing down yourself down when falling, so you don't hit the ground at full speed (as near to 0m/s as possible, at the very least you shouldn't be injured by landing) (e.g. jumping from higher up)

                  • Levitating yourself / other objects, so they don't move in any direction

                  • Bonus points: still won't move if influenced by another force (e.g. a thrown stone)

                  • Pushing objects out of your way (e.g. a locked door)

                  • Ability to combine multiple "gravity fields", so you could simultaneously keep a cup in place and rotate it upwards-down, while still keeping the liquid inside



                  Gravity is the measurement of the degree to which mass-energy curves space time around it. What you describe above is a bunch of stuff that seems convincing to a human if you described gravity magic to.



                  A physics based process won't neatly fit in a "stuff that X magic would do" that a human would narrate, because physics doesn't really care what a human finds narratively convincing.



                  As an example, Electro-Magnitism covers everything from being able to touch things, see things, shoot lightning, and pretty much all of chemistry. Meanwhile, the human "story" based Magnatism is about being able to move metals around (because they are "magnetic"), and Electricty magic is about lightning bolts and static electricty.



                  If you want anthropomorphic magic, magic that makes sense mythologically or in categories humans care about, it has to be powered by an intelligence with a similar mind to ours. That intelligence somehow set up the rules in ways that we find are pleasing.



                  Spells become "instructions" to this intelligence, who then does what asked. The being being interacted with could be a god-like entity, a demon, or a post-singularity AI, or an ancient forgotten society of super-scientists who left "macros" active that their children used to play with.



                  In the last case, the affinity of certain people for certain kinds of magic is akin to being able to unlock a certain smart phone with your voice, because it coincidentally is similar to the real owner (it doesn't have to be voice-locked, but you can imagine it being so). The fact that these ancient intelligences let their children play with fireballs would be explained by the fact that everyone in their society was backed up, and repair/resurrection was trivial; that technology doesn't recognize humans as being in their pattern-banks. Humans who can heal are actually using the "veternarian" macros.



                  Now, once we divorce the physical mechanism from the "kind of magic", the fact that these are "gravity" macros is because they are the kind of thing that the parent programmed for their kid (or maybe bought the "play with gravity" kit for their kid's system).



                  Your entire environment could be a simulation running in an ancient computer, with the original hosts long exinct. The matter you are walking on and breathing could be programmable computronium (mass-energy optimized to do computation) that emulates being matter so as to leave a "natural" experience for growing up, but being computronium this is just an option; "magic" literally ignores the apparent laws of physics, because the laws of physics as you experience them are just an emulation layer.



                  Being able to float in either of these cases is little more than a matter of changing the position value of a variable in a specific coordinate space. Now, no human has access to this underlying system; so they cannot insert the wrong arguments and cause the water on the earth to start to orbit the other way around the sun.







                  share|improve this answer












                  share|improve this answer



                  share|improve this answer










                  answered yesterday









                  YakkYakk

                  8,84911238




                  8,84911238











                  • $begingroup$
                    "There is no spoon"
                    $endgroup$
                    – EveryBitHelps
                    12 hours ago
















                  • $begingroup$
                    "There is no spoon"
                    $endgroup$
                    – EveryBitHelps
                    12 hours ago















                  $begingroup$
                  "There is no spoon"
                  $endgroup$
                  – EveryBitHelps
                  12 hours ago




                  $begingroup$
                  "There is no spoon"
                  $endgroup$
                  – EveryBitHelps
                  12 hours ago











                  2












                  $begingroup$

                  I'm borrowing heavily from the "Irregular at the Magic Highschool" light novel series where magic is done by influencing the information bodies of particles. Basically assume everything has an "information body" similar to how in physics simulations every particle has some sort of matrix/vector which stores its properties. These properties can be position, time, velocity, mass, energy, etc... This is somewhat similar to string theory where particles are strings and properties of a particle are determined by the vibration of the string.



                  Whichever way you want to go about it, gravity magic influences the vibration/information of the particle related to gravity. There are no "gravity fields" in this solution so I can't check off the ability to create multiple gravity fields but you can still rotate a cup and not have any liquid fall out by influencing the particles of the cup along with the liquid inside.






                  share|improve this answer









                  $endgroup$

















                    2












                    $begingroup$

                    I'm borrowing heavily from the "Irregular at the Magic Highschool" light novel series where magic is done by influencing the information bodies of particles. Basically assume everything has an "information body" similar to how in physics simulations every particle has some sort of matrix/vector which stores its properties. These properties can be position, time, velocity, mass, energy, etc... This is somewhat similar to string theory where particles are strings and properties of a particle are determined by the vibration of the string.



                    Whichever way you want to go about it, gravity magic influences the vibration/information of the particle related to gravity. There are no "gravity fields" in this solution so I can't check off the ability to create multiple gravity fields but you can still rotate a cup and not have any liquid fall out by influencing the particles of the cup along with the liquid inside.






                    share|improve this answer









                    $endgroup$















                      2












                      2








                      2





                      $begingroup$

                      I'm borrowing heavily from the "Irregular at the Magic Highschool" light novel series where magic is done by influencing the information bodies of particles. Basically assume everything has an "information body" similar to how in physics simulations every particle has some sort of matrix/vector which stores its properties. These properties can be position, time, velocity, mass, energy, etc... This is somewhat similar to string theory where particles are strings and properties of a particle are determined by the vibration of the string.



                      Whichever way you want to go about it, gravity magic influences the vibration/information of the particle related to gravity. There are no "gravity fields" in this solution so I can't check off the ability to create multiple gravity fields but you can still rotate a cup and not have any liquid fall out by influencing the particles of the cup along with the liquid inside.






                      share|improve this answer









                      $endgroup$



                      I'm borrowing heavily from the "Irregular at the Magic Highschool" light novel series where magic is done by influencing the information bodies of particles. Basically assume everything has an "information body" similar to how in physics simulations every particle has some sort of matrix/vector which stores its properties. These properties can be position, time, velocity, mass, energy, etc... This is somewhat similar to string theory where particles are strings and properties of a particle are determined by the vibration of the string.



                      Whichever way you want to go about it, gravity magic influences the vibration/information of the particle related to gravity. There are no "gravity fields" in this solution so I can't check off the ability to create multiple gravity fields but you can still rotate a cup and not have any liquid fall out by influencing the particles of the cup along with the liquid inside.







                      share|improve this answer












                      share|improve this answer



                      share|improve this answer










                      answered 17 hours ago









                      CptLaskyCptLasky

                      414




                      414





















                          1












                          $begingroup$

                          This is more of an addition to L.Dutch♦'s answer, controlling magnitude and sign of the masses involved in the gravitational interaction, but to achieve some of the points you mentioned (pushing a door out of the way, stopping movement even when affected by an outside force) you need to have a solution for sideways motion and rotation.



                          For sideways motion you can take into account the fact that Earth isn't a point source for gravity, instead we are pulled towards each particle that makes up the Earth and our gravity vector is the sum of all those forces. If your character can control the effect of gravity from different sources separately, then sideways motion can be achieved by getting a push from one half of the Earth and getting pulled by the other. You could even use gravity from other celestial bodies (Sun, Jupiter, supermassive black hole at the center of the the Milky Way).



                          Rotation can be achieved as described in Whitecold's comment, with nonuniform gravity: have half of the thing you're rotating be pulled down and the other half pulled up (gently so you don't tear it apart). Your rotating cup of water that doesn't spill could be done by having gravity push "into" the cup just near the mouth, pushing from the bottom of the cup to counteract the movement the first push would impart, all the while keeping the cup and water in the air (this might require more skill from your character than simple levitation).






                          share|improve this answer








                          New contributor




                          JCP is a new contributor to this site. Take care in asking for clarification, commenting, and answering.
                          Check out our Code of Conduct.






                          $endgroup$

















                            1












                            $begingroup$

                            This is more of an addition to L.Dutch♦'s answer, controlling magnitude and sign of the masses involved in the gravitational interaction, but to achieve some of the points you mentioned (pushing a door out of the way, stopping movement even when affected by an outside force) you need to have a solution for sideways motion and rotation.



                            For sideways motion you can take into account the fact that Earth isn't a point source for gravity, instead we are pulled towards each particle that makes up the Earth and our gravity vector is the sum of all those forces. If your character can control the effect of gravity from different sources separately, then sideways motion can be achieved by getting a push from one half of the Earth and getting pulled by the other. You could even use gravity from other celestial bodies (Sun, Jupiter, supermassive black hole at the center of the the Milky Way).



                            Rotation can be achieved as described in Whitecold's comment, with nonuniform gravity: have half of the thing you're rotating be pulled down and the other half pulled up (gently so you don't tear it apart). Your rotating cup of water that doesn't spill could be done by having gravity push "into" the cup just near the mouth, pushing from the bottom of the cup to counteract the movement the first push would impart, all the while keeping the cup and water in the air (this might require more skill from your character than simple levitation).






                            share|improve this answer








                            New contributor




                            JCP is a new contributor to this site. Take care in asking for clarification, commenting, and answering.
                            Check out our Code of Conduct.






                            $endgroup$















                              1












                              1








                              1





                              $begingroup$

                              This is more of an addition to L.Dutch♦'s answer, controlling magnitude and sign of the masses involved in the gravitational interaction, but to achieve some of the points you mentioned (pushing a door out of the way, stopping movement even when affected by an outside force) you need to have a solution for sideways motion and rotation.



                              For sideways motion you can take into account the fact that Earth isn't a point source for gravity, instead we are pulled towards each particle that makes up the Earth and our gravity vector is the sum of all those forces. If your character can control the effect of gravity from different sources separately, then sideways motion can be achieved by getting a push from one half of the Earth and getting pulled by the other. You could even use gravity from other celestial bodies (Sun, Jupiter, supermassive black hole at the center of the the Milky Way).



                              Rotation can be achieved as described in Whitecold's comment, with nonuniform gravity: have half of the thing you're rotating be pulled down and the other half pulled up (gently so you don't tear it apart). Your rotating cup of water that doesn't spill could be done by having gravity push "into" the cup just near the mouth, pushing from the bottom of the cup to counteract the movement the first push would impart, all the while keeping the cup and water in the air (this might require more skill from your character than simple levitation).






                              share|improve this answer








                              New contributor




                              JCP is a new contributor to this site. Take care in asking for clarification, commenting, and answering.
                              Check out our Code of Conduct.






                              $endgroup$



                              This is more of an addition to L.Dutch♦'s answer, controlling magnitude and sign of the masses involved in the gravitational interaction, but to achieve some of the points you mentioned (pushing a door out of the way, stopping movement even when affected by an outside force) you need to have a solution for sideways motion and rotation.



                              For sideways motion you can take into account the fact that Earth isn't a point source for gravity, instead we are pulled towards each particle that makes up the Earth and our gravity vector is the sum of all those forces. If your character can control the effect of gravity from different sources separately, then sideways motion can be achieved by getting a push from one half of the Earth and getting pulled by the other. You could even use gravity from other celestial bodies (Sun, Jupiter, supermassive black hole at the center of the the Milky Way).



                              Rotation can be achieved as described in Whitecold's comment, with nonuniform gravity: have half of the thing you're rotating be pulled down and the other half pulled up (gently so you don't tear it apart). Your rotating cup of water that doesn't spill could be done by having gravity push "into" the cup just near the mouth, pushing from the bottom of the cup to counteract the movement the first push would impart, all the while keeping the cup and water in the air (this might require more skill from your character than simple levitation).







                              share|improve this answer








                              New contributor




                              JCP is a new contributor to this site. Take care in asking for clarification, commenting, and answering.
                              Check out our Code of Conduct.









                              share|improve this answer



                              share|improve this answer






                              New contributor




                              JCP is a new contributor to this site. Take care in asking for clarification, commenting, and answering.
                              Check out our Code of Conduct.









                              answered yesterday









                              JCPJCP

                              111




                              111




                              New contributor




                              JCP is a new contributor to this site. Take care in asking for clarification, commenting, and answering.
                              Check out our Code of Conduct.





                              New contributor





                              JCP is a new contributor to this site. Take care in asking for clarification, commenting, and answering.
                              Check out our Code of Conduct.






                              JCP is a new contributor to this site. Take care in asking for clarification, commenting, and answering.
                              Check out our Code of Conduct.





















                                  1












                                  $begingroup$

                                  The Higgs field gives elementary particles their mass, and having some sort of a mechanism to attenuate or disable its interactions could work to your benefit at a level that doesn't really have a reasonably obvious counterargument. So in effect, the magic could somehow stop this interaction, making object momentarily more or less massive. (The remote pushing effect you mentioned I found the most difficult to explain...) The intricacies work very much differently for charged leptons and force carrier bosons, though.



                                  Most of the mass of matter is from the mass of the atomic nuclei, and ≈99% of that mass doesn't come from the mass of the individual elementary particles but the strong force; potential energies of the quarks (and hence protons and neutrons) bound together can be translated to mass via E=mc^2. Another explanation for your magic could thus be not exactly one that works by meddling with gravity but meddling with another fundamental force which affects mass which in turn gets picked up unmodified gravity. The strong force is mediated by gluons (analogous to photons for the electromagnetic interactions), so off we go with gluemancy.



                                  The problem with this all is of course, that if one is capable enough to modify these tine, fundamental properties of particles, it doesn't easily make much sense to be able to only demonstrate gravity-based powers.






                                  share|improve this answer








                                  New contributor




                                  JoonasD6 is a new contributor to this site. Take care in asking for clarification, commenting, and answering.
                                  Check out our Code of Conduct.






                                  $endgroup$

















                                    1












                                    $begingroup$

                                    The Higgs field gives elementary particles their mass, and having some sort of a mechanism to attenuate or disable its interactions could work to your benefit at a level that doesn't really have a reasonably obvious counterargument. So in effect, the magic could somehow stop this interaction, making object momentarily more or less massive. (The remote pushing effect you mentioned I found the most difficult to explain...) The intricacies work very much differently for charged leptons and force carrier bosons, though.



                                    Most of the mass of matter is from the mass of the atomic nuclei, and ≈99% of that mass doesn't come from the mass of the individual elementary particles but the strong force; potential energies of the quarks (and hence protons and neutrons) bound together can be translated to mass via E=mc^2. Another explanation for your magic could thus be not exactly one that works by meddling with gravity but meddling with another fundamental force which affects mass which in turn gets picked up unmodified gravity. The strong force is mediated by gluons (analogous to photons for the electromagnetic interactions), so off we go with gluemancy.



                                    The problem with this all is of course, that if one is capable enough to modify these tine, fundamental properties of particles, it doesn't easily make much sense to be able to only demonstrate gravity-based powers.






                                    share|improve this answer








                                    New contributor




                                    JoonasD6 is a new contributor to this site. Take care in asking for clarification, commenting, and answering.
                                    Check out our Code of Conduct.






                                    $endgroup$















                                      1












                                      1








                                      1





                                      $begingroup$

                                      The Higgs field gives elementary particles their mass, and having some sort of a mechanism to attenuate or disable its interactions could work to your benefit at a level that doesn't really have a reasonably obvious counterargument. So in effect, the magic could somehow stop this interaction, making object momentarily more or less massive. (The remote pushing effect you mentioned I found the most difficult to explain...) The intricacies work very much differently for charged leptons and force carrier bosons, though.



                                      Most of the mass of matter is from the mass of the atomic nuclei, and ≈99% of that mass doesn't come from the mass of the individual elementary particles but the strong force; potential energies of the quarks (and hence protons and neutrons) bound together can be translated to mass via E=mc^2. Another explanation for your magic could thus be not exactly one that works by meddling with gravity but meddling with another fundamental force which affects mass which in turn gets picked up unmodified gravity. The strong force is mediated by gluons (analogous to photons for the electromagnetic interactions), so off we go with gluemancy.



                                      The problem with this all is of course, that if one is capable enough to modify these tine, fundamental properties of particles, it doesn't easily make much sense to be able to only demonstrate gravity-based powers.






                                      share|improve this answer








                                      New contributor




                                      JoonasD6 is a new contributor to this site. Take care in asking for clarification, commenting, and answering.
                                      Check out our Code of Conduct.






                                      $endgroup$



                                      The Higgs field gives elementary particles their mass, and having some sort of a mechanism to attenuate or disable its interactions could work to your benefit at a level that doesn't really have a reasonably obvious counterargument. So in effect, the magic could somehow stop this interaction, making object momentarily more or less massive. (The remote pushing effect you mentioned I found the most difficult to explain...) The intricacies work very much differently for charged leptons and force carrier bosons, though.



                                      Most of the mass of matter is from the mass of the atomic nuclei, and ≈99% of that mass doesn't come from the mass of the individual elementary particles but the strong force; potential energies of the quarks (and hence protons and neutrons) bound together can be translated to mass via E=mc^2. Another explanation for your magic could thus be not exactly one that works by meddling with gravity but meddling with another fundamental force which affects mass which in turn gets picked up unmodified gravity. The strong force is mediated by gluons (analogous to photons for the electromagnetic interactions), so off we go with gluemancy.



                                      The problem with this all is of course, that if one is capable enough to modify these tine, fundamental properties of particles, it doesn't easily make much sense to be able to only demonstrate gravity-based powers.







                                      share|improve this answer








                                      New contributor




                                      JoonasD6 is a new contributor to this site. Take care in asking for clarification, commenting, and answering.
                                      Check out our Code of Conduct.









                                      share|improve this answer



                                      share|improve this answer






                                      New contributor




                                      JoonasD6 is a new contributor to this site. Take care in asking for clarification, commenting, and answering.
                                      Check out our Code of Conduct.









                                      answered yesterday









                                      JoonasD6JoonasD6

                                      111




                                      111




                                      New contributor




                                      JoonasD6 is a new contributor to this site. Take care in asking for clarification, commenting, and answering.
                                      Check out our Code of Conduct.





                                      New contributor





                                      JoonasD6 is a new contributor to this site. Take care in asking for clarification, commenting, and answering.
                                      Check out our Code of Conduct.






                                      JoonasD6 is a new contributor to this site. Take care in asking for clarification, commenting, and answering.
                                      Check out our Code of Conduct.



























                                          draft saved

                                          draft discarded
















































                                          Thanks for contributing an answer to Worldbuilding Stack Exchange!


                                          • Please be sure to answer the question. Provide details and share your research!

                                          But avoid


                                          • Asking for help, clarification, or responding to other answers.

                                          • Making statements based on opinion; back them up with references or personal experience.

                                          Use MathJax to format equations. MathJax reference.


                                          To learn more, see our tips on writing great answers.




                                          draft saved


                                          draft discarded














                                          StackExchange.ready(
                                          function ()
                                          StackExchange.openid.initPostLogin('.new-post-login', 'https%3a%2f%2fworldbuilding.stackexchange.com%2fquestions%2f141533%2fgravity-magic-how-does-it-work%23new-answer', 'question_page');

                                          );

                                          Post as a guest















                                          Required, but never shown





















































                                          Required, but never shown














                                          Required, but never shown












                                          Required, but never shown







                                          Required, but never shown

































                                          Required, but never shown














                                          Required, but never shown












                                          Required, but never shown







                                          Required, but never shown







                                          Popular posts from this blog

                                          Adding axes to figuresAdding axes labels to LaTeX figuresLaTeX equivalent of ConTeXt buffersRotate a node but not its content: the case of the ellipse decorationHow to define the default vertical distance between nodes?TikZ scaling graphic and adjust node position and keep font sizeNumerical conditional within tikz keys?adding axes to shapesAlign axes across subfiguresAdding figures with a certain orderLine up nested tikz enviroments or how to get rid of themAdding axes labels to LaTeX figures

                                          Luettelo Yhdysvaltain laivaston lentotukialuksista Lähteet | Navigointivalikko

                                          Gary (muusikko) Sisällysluettelo Historia | Rockin' High | Lähteet | Aiheesta muualla | NavigointivalikkoInfobox OKTuomas "Gary" Keskinen Ancaran kitaristiksiProjekti Rockin' High